Forums

Chess Puzzle

Sort:
xan701

you can put the rooks on too

williamscothorn1106
rooperi wrote:

try again.... I'm not sure this position can be reached?

 


 poop

shoopi

Can anyone actually create an illegal position with 27 pieces where no pieces is attacked by another?

I can't. This puzzle is fiendishly difficult.

Tyzer

Ahahaha, it's this puzzle again (after more than a year!). I spent ages working out that 26-piece configuration...I shall leave it as a challenge for anyone to best that. (I honestly can't think of a way to put rooks on it, legal or not.) I'm starting to wonder whether the OP really has the solution to this puzzle, or whether it was a simple (but fun!) exercise in trolling.

(Well okay, if you allow illegal positions you can do stupid stuff like this

 

 

which is 30 pieces (okay, I can squeeze in another two knights if I use an FEN to drop the kings completely, but this is already silly enough). Though this is illegal not only in terms of positions not reachable in a game, but also by having more knights than is legally possible.)

shoopi

Tyzer, that is correct, thanks. I was thinking more like too many captures illegal, but anyway, I can't even crack any illegal position with a rook in it!

I also didn't notice this thread is more than a year old, and the solution was never given. I think that the maximum is 26 pieces indeed.

nicobro999

nicobro999
Tyzer wrote:

Ahahaha, it's this puzzle again (after more than a year!). I spent ages working out that 26-piece configuration...I shall leave it as a challenge for anyone to best that. (I honestly can't think of a way to put rooks on it, legal or not.) I'm starting to wonder whether the OP really has the solution to this puzzle, or whether it was a simple (but fun!) exercise in trolling.

(Well okay, if you allow illegal positions you can do stupid stuff like this

 

 

which is 30 pieces (okay, I can squeeze in another two knights if I use an FEN to drop the kings completely, but this is already silly enough). Though this is illegal not only in terms of positions not reachable in a game, but also by having more knights than is legally possible.)


Well... and then you can put bishops on the white squares in between the knights which would make it 37... so im guessing thats illegal... lol 

planeden

i got 25?  thanks to jeremy as a starting point. nevermind. 

nicobro999

davidan
jeremygs7 wrote:

Good question. I got 24 and it's also symmetry.

 

 


add the rocks on g1 and e1 followed by the queen on a1 and that way with u getg 30 pieces whats left are the two bishops figure it out for yourself

ILuvPawn

Here's a 26-piece legal position that fulfils all the original criteria that has a rook in it to boot. The empty square on f1 is begging for a promoted White knight on it but the Black pawn on e3 doesn't allow for it. It wouldn't surprise me if someone could reconfigure this to fit a White pawn or a promoted knight in there somewhere.

Tyzer
nicobro999 wrote:
Tyzer wrote:

Ahahaha, it's this puzzle again (after more than a year!). I spent ages working out that 26-piece configuration...I shall leave it as a challenge for anyone to best that. (I honestly can't think of a way to put rooks on it, legal or not.) I'm starting to wonder whether the OP really has the solution to this puzzle, or whether it was a simple (but fun!) exercise in trolling.

(Well okay, if you allow illegal positions you can do stupid stuff like this

 

 

which is 30 pieces (okay, I can squeeze in another two knights if I use an FEN to drop the kings completely, but this is already silly enough). Though this is illegal not only in terms of positions not reachable in a game, but also by having more knights than is legally possible.)


Well... and then you can put bishops on the white squares in between the knights which would make it 37... so im guessing thats illegal... lol 


Actually, every unoccupied square on the board is defended by (multiple) knights or the kings, so no, it's not possible to put bishops there.

senorfrog64

why can't it be 32, from starting legal position? your solutions assume some moves, but even with a kings pawn start, theres no attacking pieces

planeden
senorfrog64 wrote:

why can't it be 32, from starting legal position? your solutions assume some moves, but even with a kings pawn start, theres no attacking pieces


the op corrected it later.  there can be no attacking or defending pieces. 

JMB2010

Here's a position with 27.

hanngo
JMB2010 wrote:

Here's a position with 27.

 

 


Correct!

shoopi
JMB2010 wrote:

Here's a position with 27.

 

 

Amazing.

I did not think it was possible... but there you go. Just, how did you come up with the solution? And how much time did it take you?

P.S. I have added a proof game to show it is a legal position:


Tyzer

Wow, very impressive!